Những câu hỏi liên quan
huynh van duong
Xem chi tiết
Lê Song Phương
Xem chi tiết
Lê Song Phương
9 tháng 4 2022 lúc 19:53

Cái đầu tiên là \(\sqrt[n]{\frac{a_1^n+a_2^n+a_3^n+...+a_n^n}{n}}\)nhé.

Bình luận (0)
 Khách vãng lai đã xóa
tth
Xem chi tiết
Sherry
Xem chi tiết
KCLH Kedokatoji
Xem chi tiết
Vũ Huy Hoàng
Xem chi tiết
l҉o҉n҉g҉ d҉z҉
Xem chi tiết
Quốc Bảo
Xem chi tiết
Kuro Kazuya
10 tháng 3 2017 lúc 18:32

\(\dfrac{a_1}{2-a_1}+\dfrac{a_2}{2-a_2}+...+\dfrac{a_n}{2-a_n}\ge\dfrac{n}{2n-1}\)

\(\Leftrightarrow\dfrac{a^2_1}{2a_1-a^2_1}+\dfrac{a^2_2}{2a_2-a^2_2}+...+\dfrac{a^2_n}{2a_n-a^2_2}\ge\dfrac{n}{2n-1}\)

Áp dụng bất đẳng thức cộng mẫu số

\(\Rightarrow\dfrac{a^2_1}{2a_1-a^2_1}+\dfrac{a^2_2}{2a_2-a^2_2}+...+\dfrac{a^2_n}{2a_n-a^2_2}\ge\dfrac{\left(a_1+a_2+...+a_n\right)^2}{2\left(a_1+a_2+...+a_n\right)-\left(a^2_1+a^2_2+...+a_n^2\right)}\)

\(\Rightarrow\dfrac{a^2_1}{2a_1-a^2_1}+\dfrac{a^2_2}{2a_2-a^2_2}+...+\dfrac{a^2_n}{2a_n-a^2_2}\ge\dfrac{1}{2-\left(a^2_1+a^2_2+...+a_n^2\right)}\)

Chứng minh rằng \(\dfrac{1}{2-\left(a^2_1+a_2^2+...+a^2_n\right)}\ge\dfrac{n}{2n-1}\)

\(\Leftrightarrow2n-1\ge n\left[2-\left(a^2_1+a^2_2+...+a^2_n\right)\right]\)

\(\Leftrightarrow2n-1\ge2n-n\left(a^2_1+a^2_2+...+a^2_n\right)\)

\(\Leftrightarrow-1\ge-n\left(a^2_1+a^2_2+...+a^2_n\right)\)

\(\Leftrightarrow1\le n\left(a^2_1+a^2_2+...+a^2_n\right)\)

\(\Leftrightarrow\dfrac{1}{n}\le a^2_1+a^2_2+...+a^2_n\)

Áp dụng bất đẳng thức cộng mẫu số

\(\Rightarrow VP=\dfrac{a^2_1}{1}+\dfrac{a^2_2}{1}+...+\dfrac{a^2_n}{1}\ge\dfrac{\left(a_1+a_2+...+a_n\right)^2}{n}=\dfrac{1}{n}\)

\(\Rightarrow\) đpcm

Vậy \(\dfrac{1}{2-\left(a^2_1+a_2^2+...+a^2_n\right)}\ge\dfrac{n}{2n-1}\)

\(\Rightarrow\dfrac{a_1}{2-a_1}+\dfrac{a_2}{2-a_2}+...+\dfrac{a_n}{2-a_n}\ge\dfrac{n}{2n-1}\) ( đpcm )

Bình luận (1)
Avicii
Xem chi tiết
Ơ Ơ BUỒN CƯỜI
28 tháng 5 2018 lúc 15:11

ÁP DỤNG BĐT Cauchy ta có : 

\(\text{a}_1+\text{a}_2+...+\text{a}_n\ge n^n\sqrt{\text{a}_1.\text{a}_2....\text{a}_n}\)  (1) 

\(\frac{1}{\text{a}_1}+\frac{1}{\text{a}_2}+...+\frac{1}{\text{a}_n}\ge n^n\sqrt{\frac{1}{\text{a}_1}\cdot\frac{1}{\text{a}_2}\cdot...\cdot\frac{1}{\text{a}_n}}\)(2) 

Nhân (1) và (2) vế với vế tương ứng ta có được BĐT (*) 

Đẳng thức xảy ra \(\Leftrightarrow\hept{\begin{cases}\text{a}_1=\text{a}_2=...=\text{a}_n\\\frac{1}{\text{a}_1}=\frac{1}{\text{a}_2}=...=\frac{1}{\text{a}_n}\end{cases}}\)

                             \(\Leftrightarrow\text{a}_1=\text{a}_2=...=\text{a}_n\)

Bình luận (0)